FIND THE MISSING NUMBER
20 - s = 11?

Answers

Answer 1
s = 9
20-11=9
Annd dna

Related Questions

For every 20 grams of fat, 180 calories are added to a serving size of food.

How many calories are added with 4 grams of fat?

Answers

Answer:

36 Calories

Step-by-step explanation:

first find what is added with ONE gram of fat.

do that by dividing

180/20

=9

then multiply by 4

9*4

=36

:)

is this wrong?? if it is pls help me choose the correct answer

(choose the answer that best corrects the misplaced modifier in each sentence. )

Answers

you didn’t show an image?..:(
image?? how’s everyone’s day?

please help im almost a bit close, i’ll mark and brainlest please be serious or i will take my points back.

shoutout:
brainly.com/app/profile/19154158/answers

Answers

Answer:

22

Step-by-step explanation:

The median shows the midpoint of the data, so for a box and whisker plot the median would be the number that splits the box. for this question the median would be 22

Answer:

22

Step-by-step explanation:

The median lands on 22

What is the area of the object above? (Picture attached)

Answers

Answer:

155 cm

rectangle 1: 5x7 = 35

rectangle 2: 8x15 = 120

35 + 120 = 155 cm

Step-by-step explanation:

155 cm Hope This Helps :)

Mr. Hamilton surveyed his class to find the total number of hours each of his students slept the previous night. The histogram shows the results of the survey.

How many more students slept 4 – 7 hours than 12 – 15 hours?

Answers

Answer:

6 more students

Step-by-step explanation:

hope this helps

Why are the lit up portions of
the of Earth and Moons (in
the inner circle of the
diagrams) all pointing the
same way?
SUNLIGHT

GIVING BRAINLIEST

Answers

because the moon is reflecting the sun’s light (i’m not sure if that is the answer you are looking for)

Select the correct location on the graph.
Identify the cluster in the data.

Answers

Answer: (5,7) (5,8)

Step-by-step explanation:

Answer: The 2nd choice

X axis: Between 3 and 6

Y axis: Between 7 and 9

Step-by-step explanation:

y/x = 8
Show wether x and y show direct variation
Help I don’t wanna go to summer school

Answers

Answer:

y/x = 8 shows DIRECT variation.

Step-by-step explanation:

y varies directly as x if y = kx for some constant, k.  In other words, y/x is constant.

just a reminder ur loved :))

Over the summer, the facility manager of a public swimming pool collected information on the amount of water in the pool and the number of swimmers. The results are shown in the scatter plot below. Which statement is supported by the data in the scatter plot?

A. As the number of swimmers increases, the amount of water in the pool tends to decrease.

B. As the number of swimmers increases, the amount of water in the pool tends to increase.

C. As the number of swimmers increases, the amount of water in the pool tends to stay the same.

D. There is no relationship between the number of swimmers and the amount of water in the pool.

Answers

The answers is B

Step-by-step explanation:

EDGE2021

As the number of swimmers increases, the amount of water in the pool tends to decrease.

Thus, option (A) is correct.

Given:

A scatter plot which shows he amount of water in the pool and the number of swimmers.

Now, from the scatter plot it can be seen that

As the number of swimmer increases there is no such change in amount of water.

There is only increase in a amount of water when the number of swimmers were between 60 to 70.

But at last the plot shows that the amount of water decrease.

Therefore, number of swimmers increases, the amount of water decrease.

Thus, option (A) is correct.

Learn more about Scatter plot here:

https://brainly.com/question/29231735

#SPJ6

Can sum1 help me with this asap pls :)

Answers

5 units (extra characters......)
the answer would be 5 units

In which expression is the coefficient of the n term -1?

Answers

Answer:

jubiau 72t

Step-by-step explanation:

and also tbhe

The answer is c -2n-n 5

Find all roots of the polynomial function f(x) = 2x^4-x^3-13x^2+5x+15

Answers

Answer:

Roots are :  1, -7 and -7

Step-by-step explanation:

When we put x = -1 in F(x) it is equal to 0

Thus, (x + 1) is factor of given function.

When we divide F(x) by (x + 1) it gives the value (x² - 6x - 7)

factorizing the work  x² - 6x - 7 by middle term splitting.

> x² - 6x - 7 = x² - 7x + x - 7

>  x(x - 7) + 1(x - 7)

>  (x + 1) (x - 7)

then,  x³ - 5x² - 13x - 7 = (x + 1)(x + 1)(x - 7)

x = -1, -1, 7 the factor.

D and C is the answers.

Step-by-step explanation:

1 -7 7


Hope this helps you out

ANSWER FOR BRAINLIEST AND PLEEASEE EXPLAIN :)

Answers

Answer: I think Value of x is 8 I’m not sure

Answer:

8

Step-by-step explanation:

the right angle is 90 degrees, the line is 180 so 180-90 is 90 so 6X+4 + 5X-2= 90 you can counter the -2 by using the +4 so it just becomes +2 90-2=88 so the answer is 8

Use the given line of best fit to approximate the rate of change relative to the scatter plot below.

A. 1/20 errors per word translated

B. 1/5 errors per word translated

C. 1/10 errors per word translated

D. 3/20 errors per word translated

Answers

Answer:

C. 1/10 errors per word translated

Step-by-step explanation:

According to this graph, we can see that every 100 words-translated, there are about 10 errors (red line). The red line continues at this slope/ratio, and so it would be 10/100 or 1/10 simplified.

The rate of change relative to the scatter plot below will be 1/10 errors per word translated. Option C is correct.

What is the line of best fit?

A mathematical notion called the line of the best fit connects points spread throughout a graph. It's a type of linear regression that uses scatter data to figure out the best way to define the dots' relationship.

[tex]\rm m = \dfrac{n\sum xy-\sum x \sum y}{n\sum x^2 - (\sum x)^2} \\\\\rm c = \dfrac{\sum y -m \sum x}{n}[/tex]

For the given condition, we have to find the rate of change relative to the scatters plot below.

The given graph is in between the total error and the translation error. This graph shows that there are around 10 translation mistakes for every 100 words (red line). Given that the red line continues at this slope and ratio, it would be expressed as 10/100 or 1/10.

Thus, the rate of change relative to the scatter plot below will be 1/10 errors per word translated. Option C is correct.

Learn more about the line of best fit here:

brainly.com/question/14279419

#SPJ6

PLS HELP ME ASAP THE PROBLEM IS IN THE PICTURE! <3​

Answers

Answer:

5 people each get 2 bags of ice cream or 2 people each get 5 bags of ice cream..

Step-by-step explanation:

Explain how to find the volume of a triangular pyramid. (Explain the steps)

Answers

Zoom in to see clearly and it shows an example there.

Mark and Carol are saving money for a trip. Carol always has $10 more dollars than Mark.

Create an equation that shows Carol's savings,c, in terms of Mark's savings, m.

PLEASE HELP I WILL DO MARK A BRAINELEST

Answers

Answer:

m+10=c

Step-by-step explanation:

Since Carol always has 10 more dollars the Mark, the equation should represents Mark's savings plus the extra $10 that Carol has.

M+10=c since 10 it’s 10 more than M

Find the volume of a cylinder. Round your answer to the nearest tenth. Do not add a comma.

B = 135.6 ft2

h = 11.2 ft

Answers

Answer:

Step-by-step explanation:

Base -- find the radius

radius -- 67.8 (135.6^2 /2)

h=11.2

Volume = 2385.59

2385.60

there are 6 boys and 5 girls in the debate class. kids are being selected to go present their work without getting opt-out where is the probability that a boy will be selected to present twice?

Answers

Answer:

30%?

Step-by-step explanation:

6 boys, 5 girls

first selection is a 6/11 chance it's a boy.

second seletion is a 6/11 chance it's a boy.

6/11 times 6/11 is... 0.2975 or about a 30% chance ( i think)

Just multiply the probability of the first event by the second. For example, if the probability of event A is 2/9 and the probability of event B is 3/9 then the probability of both events happening at the same time is (2/9)*(3/9) = 6/81 = 2/27.

HELP?! PLEASE?!
- Melanie started working at Zane's Car Wash. She recorded the number of car washes for each day during the months of October and November, and she displayed the results in the box plots shown below. Which statements best supports the information from the graph? *

Answers

Answer:

she washed more cars in October

Step-by-step explanation:

in November she washed 17 to 19 cars

in October she washed 22 to 24 cars

22 - 17 = 5 24 - 19 = 5

so she washed 5 more cars in October

plz mark brainliest if this helped you!!

5
why? i don’t feel like explaining

How many mg of a metal containing 38 percent nickel must be combined with 5 mg of pure nickel to form an alloy containing 68 percent nickel? Which of the following equations could be used to solve this?

Answers

Answer:

the Answer is C

Step-by-step explanation:

The metal needs to be 16/3 mg in weight to get the required alloy.

We can use the third equation,

(a(0.38) + 5(1.00))/(a+5) = 68/100

to solve the problem.

What do we mean by Percentage?

A percentage is a ratio where a number is shown as a fraction with respect to 100 parts of it. To remove the percentage sign we divide the number by 100.

How do we solve the given problem?

We are asked to mix a certain quantity of metal in mg containing 38 percent of nickel to 5 mg of pure nickel such that the new alloy contains 68 percent of nickel.

Let the quantity of metal taken to be 'a' mg.

∴ Nickel provided by the metal is 38% of a,

or, (38/100)*a

or, a(0.38)

We took 5 mg of pure nickel so it is 100 percent pure,

∴ Nickel in the alloy from pure nickel = 100% of 5

or, (100/100)*5

or, 5(1.00)

Total weight of the alloy = a + 5

Total nickel in the alloy = a(0.38) + 5(1.00)

We are said that the alloy constitutes 68 percent of nickel,

∴ (a(0.38) + 5(1.00))/(a+5) = 68%

or, (a(0.38) + 5(1.00))/(a+5) = 68/100

This is the equation needed to find the quantity of metal used to make the alloy. It is the same as option 3.

On solving the equation we get that the required quantity of the metal is 16/3 mg.

Learn more about Percentages at

https://brainly.com/question/843074

#SPJ2

DONT REPORT MY QUESTION ITS FOR MY SCIENCE Experiment

would any of you wonderful people here be interested in answering a few questions on this go.ogle forms quiz....

Answers

Answer:

nah am good thanks anyway

uhmm...help please :< i'll give brainliest to whoever answers all three

Answers

1. 57 is the constant rate of change

2. The ratio for Ms. Bojorquez is 32:28 which simplified is 8:7. The ratio for Ms. William is 104:98 which simplified is 52:49. So they are not equivalent ratios.

3. No, it will take almost 10 minutes.

Hope this helps!!

Can someone please help me figure out how to solve this?

Answers

Answer:

B. 48m²

Step-by-step explanation:

The formula for volume is length (base) times width times height, or LWH. If you break the shape into three separate areas, you can figure out the volume for each part and then add it up.

The large part on the bottom is 7m long, 2m across. Since we know the entire shape is 4m tall and the indent is 2m deep, we know that the height for the bottom part will also be 2m. 7 · 2 · 2 = 28.

The block on the left is 2m tall, 2m across, and its width is also 2m as shown by the measurement on the bottom-right. 2 · 2 · 2 = 8.

Finally, the block on the right is 2m tall, 3m across, with a width of 2m. 3 · 2 · 2 = 12. And when you add it all up, 28 + 8 + 12 = 48. Hope this helps! Feel free to ask any questions!

On some graph paper, graph these points in a coordinate plane: (-2, 3), (2, 3), (2, -3), (-2, -3). Connect all the points. Describe the figure that you see.
plz no links

Answers

Awnser is in the photo like fr?

Which of the data sets below have the following five-number summary which could be used to create a box plot for the data set? Select all that apply.

16 ~ 19 ~ 23 ~ 31 ~ 32

A.
16, 18, 19, 23, 23, 30, 32, 32

B.
16, 18, 19, 23, 29, 31, 32

C.
16, 19, 22, 24, 31, 32

D.
16, 18, 20, 20, 23, 26, 30, 32, 32

E.
16, 18, 20, 23, 30, 32, 32

Answers

B,D,E I think
Hope it helps
B,E,D I think I hope this was helpful and message me if it was or you can say thank you lol

Truman is buying a teddy bear and flowers for his girIfriend. Flowers Galore charges $13 for the teddy bear and $0.75 per flower. Famous Florist charges $9 for the teddy bear and $1.25 per flower. Which inequality represents the number of flowers, f, Truman would need to buy to make Flowers Galore the cheaper option?

Answers

the answers is either f < 2 or f > 2

The inequality that would make Flowers Galore the cheaper option is 13 + 0.75f < 9 + 1.25f

How to determine the inequality?

At Flowers Galore, we have:

Teddy bear = $13

Flower = $0.75

At Famous Florist, we have:

Teddy bear = $9

Flower = $1.25

So, the expressions are:

Flowers Galore: 13 + 0.75f

Famous Florist: 9 + 1.25f

When Flowers Galore is the cheaper option, we have:

13 + 0.75f < 9 + 1.25f

Hence, the inequality that would make Flowers Galore the cheaper option is 13 + 0.75f < 9 + 1.25f

Read more about inequalities at:

https://brainly.com/question/11234618

please answer this for me asap?

Answers

Answer:trapezoid 3 is the only right one

rectangle 1 and 2 are correct

parallelogram 3 is only correct

Step-by-step explanation:

sorry if i miss the first one

A bottle holds 255 milliliters of shampoo. How many liters will 32 bottles hold?

816 liters
8.16 liters
8,160 liters
81.6 liters

Answers

Answer:

816 liters

Step-by-step explanation:

Hope this helps :)

Point X is on a number line halfway between `-5` and `11`. What integer would represent point X?

Answers

the answer would be -2.2

Answer:

3 i think because  4 5 6 7 8 9 10 = 7 and  2, 1, 0, -1, -2, -3, -4, = 7

Step-by-step explanation:

And also you never put anwer to choose from.

Y ∈ [ 0, 11 ) assuming -5 and 11 are not included, as the word "between" suggests.

Other Questions
I got so many points, so i wanna give some away!!Becuase I dont want to get flagged awnser this EASY math problem6 - 1 = ___ Help please this is geometry I need help with this awnser.I will mark brainy Name all pairscomplementary angles Human environment is greatly influenced by (8.8A) Which star is hotter and brighter than our sun?someone help me please The list price of an item is $550 and it is chain discount by 50%less 10% less 20%. How much is the discount?$352$198$440$110 PLEASE DON'T SEND A LINKJayna has 4 feet of string to make toys for her cats. She wants to use the same amount of string for each toy. Does she have enough string to make the following toys? Mark all that apply. A. 3 toys that each use 3/4 foot of string B. 4 toys that each use 11/12 foot of stringC. 5 toys that each use 7/8 foot of string D. 6 toys that each use 5/6 foot of string E. 8 toys that each use 1/2 foot of string Which two compounds of oxygen are small molecules?Explain why metals conduct electricity. Refer to structure and bonding in your answer Les clbritsMatch these famous people with their professions.1.Donald Trump2.Claude Monet3.Paul Mitchell4.Dr. Phil C. McGraw5.Serena Williams6.Barbara Walters7.Beethoven8.Johnny Cochran Question 15: what is the message of the D tour and Connor speech? Question 16: what rhetorical appeal does Connor use in this quoted portion of his address? A. Ethos B. PathosC. logos 32 points help asap please A bag contains pennies, nickels, dimes, and quarters. There are 50 coins in all. Of the coins, 8% are pennies and 36% are dimes. There are 9 more nickels than pennies. How much money does the bag contain? Unusual artic weather blasted Houston, Texas on Sunday, February 14, 2021.are the commas in the correct place for this. drag and drop each number to its correct position on the number line -3/8, 7/8, -2 1/2, 2 1/2 I NEED HELPP FASTTT 1.______ est-ce que vous allez faire aprs le cours?2.______muses vas-tu visiter?3.______est-ce qu'on dne ce soir? If p and q are co-interior angles, write the relation between them The domain of the function d(t)= 1/3t - 12 is (9, 12, 15, 18, 21). What is the range? Which point is a solution to y is less than or equal to 4x + 5? empathy is the same as sympathy. true or false Joel wants to buy a new tablet computer from a store having a 20% off sale on all tablets. The tablet he wants has an original cost of $190. He also wants to make sure he has enough money to pay the 5% sales tax.Which expression can be used to calculate the cost of the tablet?A. (0.20)(190)(0.05)B. (0.80)(190)(0.05)C. (0.20)(190)(1.05)D. (0.80)(190)(1.05)